376 lines
16 KiB
TeX
376 lines
16 KiB
TeX
\documentclass[10pt, a4paper]{exam}
|
|
\printanswers % Comment this line to hide the answers
|
|
\usepackage[utf8]{inputenc}
|
|
\usepackage[T1]{fontenc}
|
|
\usepackage[ngerman]{babel}
|
|
\usepackage{listings}
|
|
\usepackage{float}
|
|
\usepackage{graphicx}
|
|
\usepackage{color}
|
|
\usepackage{listings}
|
|
\usepackage[dvipsnames]{xcolor}
|
|
\usepackage{tabularx}
|
|
\usepackage{geometry}
|
|
\usepackage{color,graphicx,overpic}
|
|
\usepackage{amsmath,amsthm,amsfonts,amssymb}
|
|
\usepackage{tabularx}
|
|
\usepackage{listings}
|
|
\usepackage[many]{tcolorbox}
|
|
\usepackage{multicol}
|
|
\usepackage{hyperref}
|
|
\usepackage{pgfplots}
|
|
\usepackage{bussproofs}
|
|
\usepackage{tikz}
|
|
\usetikzlibrary{automata, arrows.meta, positioning}
|
|
\renewcommand{\solutiontitle}{\noindent\textbf{Antwort}: }
|
|
\SolutionEmphasis{\small}
|
|
\geometry{top=1cm,left=1cm,right=1cm,bottom=1cm}
|
|
|
|
\pdfinfo{
|
|
/Title (Grundlagen und diskrete Strukturen - Prüfungsvorbereitung)
|
|
/Creator (TeX)
|
|
/Producer (pdfTeX 1.40.0)
|
|
/Author (Robert Jeutter)
|
|
/Subject ()
|
|
}
|
|
\title{Grundlagen und diskrete Strukturen - Prüfungsvorbereitung}
|
|
\author{}
|
|
\date{}
|
|
|
|
% Don't print section numbers
|
|
\setcounter{secnumdepth}{0}
|
|
|
|
\newtcolorbox{myboxii}[1][]{
|
|
breakable,
|
|
freelance,
|
|
title=#1,
|
|
colback=white,
|
|
colbacktitle=white,
|
|
coltitle=black,
|
|
fonttitle=\bfseries,
|
|
bottomrule=0pt,
|
|
boxrule=0pt,
|
|
colframe=white,
|
|
overlay unbroken and first={
|
|
\draw[red!75!black,line width=3pt]
|
|
([xshift=5pt]frame.north west) --
|
|
(frame.north west) --
|
|
(frame.south west);
|
|
\draw[red!75!black,line width=3pt]
|
|
([xshift=-5pt]frame.north east) --
|
|
(frame.north east) --
|
|
(frame.south east);
|
|
},
|
|
overlay unbroken app={
|
|
\draw[red!75!black,line width=3pt,line cap=rect]
|
|
(frame.south west) --
|
|
([xshift=5pt]frame.south west);
|
|
\draw[red!75!black,line width=3pt,line cap=rect]
|
|
(frame.south east) --
|
|
([xshift=-5pt]frame.south east);
|
|
},
|
|
overlay middle and last={
|
|
\draw[red!75!black,line width=3pt]
|
|
(frame.north west) --
|
|
(frame.south west);
|
|
\draw[red!75!black,line width=3pt]
|
|
(frame.north east) --
|
|
(frame.south east);
|
|
},
|
|
overlay last app={
|
|
\draw[red!75!black,line width=3pt,line cap=rect]
|
|
(frame.south west) --
|
|
([xshift=5pt]frame.south west);
|
|
\draw[red!75!black,line width=3pt,line cap=rect]
|
|
(frame.south east) --
|
|
([xshift=-5pt]frame.south east);
|
|
},
|
|
}
|
|
|
|
\begin{document}
|
|
\begin{myboxii}[Disclaimer]
|
|
Aufgaben aus dieser Vorlage stammen aus der Vorlesung \textit{Grundlagen und diskrete Strukturen} und wurden zu Übungszwecken verändert oder anders formuliert! Für die Korrektheit der Lösungen wird keine Gewähr gegeben.
|
|
\end{myboxii}
|
|
|
|
Erlaubte Hilfsmittel: eine math. Formelsammlung/Nachschlagwerk, ein handbeschriebenes A4-Blatt mit Formeln und Ergebnissen aus der Vorlesung.
|
|
|
|
%##########################################
|
|
\begin{questions}
|
|
\question
|
|
\begin{parts}
|
|
\part Untersuche, welche der folgenden aussagenlogischen Ausdrücke logisch äquivalent sind. Begründe die Entscheidung.\\\begin{center}
|
|
$\varphi=p\rightarrow (q\wedge\overline{r})$, $\psi=(p\rightarrow q)\wedge(r\rightarrow\overline{p})$, $y=(\overline{p}\vee q)\leftrightarrow r$ \end{center}
|
|
\begin{solution}
|
|
|
|
$\varphi=p\rightarrow (q\wedge\overline{r})$
|
|
\begin{tabular}{c|c|c|c|c}
|
|
$p$ & $q$ & $r$ & $q\wedge\overline{r}$ & $p\rightarrow(q\wedge\overline{r})$ \\\hline
|
|
0 & 0 & 0 & 0 & 1 \\
|
|
0 & 0 & 1 & 0 & 1 \\
|
|
0 & 1 & 0 & 1 & 1 \\
|
|
0 & 1 & 1 & 0 & 1 \\
|
|
1 & 0 & 0 & 0 & 0 \\
|
|
1 & 0 & 1 & 1 & 1 \\
|
|
1 & 1 & 0 & 1 & 1 \\
|
|
1 & 1 & 1 & 0 & 0
|
|
\end{tabular}
|
|
|
|
$\psi=(p\rightarrow q)\wedge(r\rightarrow\overline{p})$
|
|
\begin{tabular}{c|c|c|c|c|c}
|
|
$p$ & $q$ & $r$ & $p\rightarrow q$ & $r\rightarrow\overline{p}$ & $(p\rightarrow q)\wedge(r\rightarrow\overline{p})$ \\\hline
|
|
0 & 0 & 0 & 1 & 1 & 1 \\
|
|
0 & 0 & 1 & 1 & 0 & 0 \\
|
|
0 & 1 & 0 & 1 & 1 & 1 \\
|
|
0 & 1 & 1 & 1 & 0 & 0 \\
|
|
1 & 0 & 0 & 0 & 1 & 0 \\
|
|
1 & 0 & 1 & 0 & 1 & 0 \\
|
|
1 & 1 & 0 & 1 & 0 & 0 \\
|
|
1 & 1 & 1 & 1 & 1 & 1
|
|
\end{tabular}
|
|
|
|
$y=(\overline{p}\vee q)\leftrightarrow r$
|
|
\begin{tabular}{c|c|c|c|c}
|
|
$p$ & $q$ & $r$ & $\overline{p}\vee q$ & $(\overline{p}\vee q)\leftrightarrow r$ \\\hline
|
|
0 & 0 & 0 & 0 & 1 \\
|
|
0 & 0 & 1 & 0 & 0 \\
|
|
0 & 1 & 0 & 1 & 0 \\
|
|
0 & 1 & 1 & 1 & 1 \\
|
|
1 & 0 & 0 & 0 & 1 \\
|
|
1 & 0 & 1 & 0 & 0 \\
|
|
1 & 1 & 0 & 0 & 1 \\
|
|
1 & 1 & 1 & 0 & 0
|
|
\end{tabular}
|
|
\end{solution}
|
|
|
|
\part Negiere die Aussage: $\forall S\in\mathbb{R}\exists m\in\mathbb{N} \forall n\in\mathbb{N}: n>m\Rightarrow a_n>S$
|
|
\begin{solution}
|
|
$\exists S\in\mathbb{R}\exists m\in\mathbb{N} \forall n\in\mathbb{N}: n>m\Rightarrow a_n < S$
|
|
\end{solution}
|
|
|
|
\part Negiere die Aussage: ,,In jeder GudS-Klausur gibt es mindestens eine Aufgabe, die von niemandem richtig gelöst wird''
|
|
\begin{solution}
|
|
,,Es gibt eine GudS-Klausur in der jemand jede Aufgabe richtig löst.''
|
|
\end{solution}
|
|
\end{parts}
|
|
|
|
\question Es seien $f,g:\mathbb{N}\rightarrow\mathbb{N}$ zwei Funktionen. Auf der Menge $\mathbb{N}$ der natürlichen Zahlen wird wie folgt eine Relation definiert: $a \sim b \leftrightarrow f(a)-f(b)=g(a)-g(b)$. Weise nach, dass $\sim$ eine Äquivalenzrelation ist. Für den konkreten Fall $f(x)=x^2+1$ und $g(x)=2x$ bestimme man die Äquivalenzklasse $[2]_{\backslash\sim}$
|
|
\begin{solution}
|
|
\end{solution}
|
|
|
|
\question
|
|
\begin{parts}
|
|
\part Bestimme mit Hilfe des euklidischen Algorithmus ganze Zahlen $a,b$, für die gilt $1=a*100+b*23$
|
|
\begin{solution}
|
|
$ggT(a,b)= a*x+b*y$
|
|
|
|
$\downarrow$: $b_i\rightarrow a_{i+1}$, $r_i\rightarrow b_{i+1}$
|
|
|
|
$\uparrow$: $x_i=y_{i+1}$, $y_i=x_{i+1}-q_i*y_{i+1}$
|
|
|
|
\begin{tabular}{c|c|c|c|c|c|c|c|c}
|
|
i & a & b & q (Teiler) & r(est) & x & y & Nebenrechnung $\downarrow$ & Nebenrechnung $\uparrow$ \\\hline
|
|
1 & 100 & 23 & 4 & 8 & 3 & -13 & $100-23*4 = 8$ & $100*3 + 23*(-1-4*3)= 300-299= 1$ \\
|
|
2 & 23 & 8 & 2 & 7 & -1 & 3 & $23-2*8=7$ & $23*-1 + 8*(1-2*(-1))=1$ \\
|
|
3 & 8 & 7 & 1 & 1 & 1 & -1 & $8-1+7=1$ & $8*1 + 7*(0-1*1)=1$ \\
|
|
4 & 7 & 1 & 7 & 0 & 0 & 1 & $7-7*1=0$ & $7*0+1*1 = 1$
|
|
\end{tabular}
|
|
|
|
Lösung: $a=3$, $b=-13$
|
|
\end{solution}
|
|
|
|
\part Bestimme mit Hilfe des euklidischen Algorithmus ganze Zahlen $a,b$, für die gilt $1=a*23+b*17$
|
|
\begin{solution}
|
|
\begin{tabular}{c|c|c|c|c|c|c}
|
|
i & a & b & q & r & x & y \\\hline
|
|
1 & 23 & 17 & 1 & 6 & 3 & $-1-1*3=-4$ \\
|
|
2 & 17 & 6 & 2 & 5 & -1 & $1-2*-1=3$ \\
|
|
3 & 6 & 5 & 1 & 1 & 1 & $0-1*1=-1$ \\
|
|
4 & 5 & 1 & 5 & 0 & 0 & 1
|
|
\end{tabular}
|
|
|
|
Lösung: $1=-3*23 -4*17 = 69-68 = 1$
|
|
\end{solution}
|
|
|
|
\part Untersuche, ob es ein multiplikativ inverses Element zu $\overline{23}$ in $\mathbb{Z}_{100}$ gibt und bestimme dieses gegebenfalls. Gebe außerdem ein nicht invertierbares Element außer $\overline{0}$ in $\mathbb{Z}_{100}$ an.
|
|
\begin{solution}
|
|
multiplikativ inverses: $a^{-1}*a=1$
|
|
|
|
die multiplikative Gruppe $\mathbb{Z}_n$ besteht aus den Elementen von $\mathbb{Z}_n$ die teilerfremd zu $n$ sind. Für jedes $a\in\mathbb{Z}_n^*$ gilt $ggt(a,n)=1$ und lässt sich als $1=a*x + n*y$ darstellen $\quad\Rightarrow\quad a^{-1} \equiv y(mod\ n)$
|
|
|
|
\begin{tabular}{c|c|c|c|c|c|c}
|
|
i & a & b & q & r & x & y \\\hline
|
|
1 & 100 & 23 & 4 & 8 & 3 & -13 \\
|
|
2 & 23 & 8 & 2 & 7 & -1 & 3 \\
|
|
3 & 8 & 7 & 1 & 1 & 1 & -1 \\
|
|
4 & 7 & 1 & 7 & 0 & 0 & 1
|
|
\end{tabular}
|
|
|
|
$1=100 * 3 - 23 * 13 \Rightarrow \overline{23}= -13 (mod\ 100)$
|
|
|
|
Alternativ: $a^{-1}*a=1 \rightarrow a^{-1}=1\backslash a \Rightarrow a^{-1}=\frac{1}{23}$
|
|
|
|
|
|
\end{solution}
|
|
\end{parts}
|
|
|
|
\question Gegeben sei die Menge $G=\{ \begin{pmatrix} 1&a&b\\0&1&c\\0&0&1 \end{pmatrix} \in\mathbb{R}^{(3,3)}\mid a,b,c\in\mathbb{R}\}$. Zeige, dass $G$ eine Gruppe bezüglich der Matrizenmultiplikation ist. Rechengesetze der Matrizenmultiplikation dürfen vorausgesetzt werden. Ist die Gruppe kommutativ? (ohne Beweis)
|
|
\begin{solution}
|
|
|
|
Eine nichtleere Menge $G$ von Elementen $a, b, c, ...$ heißt Gruppe, wenn in ihr eine Operation $\circ$ erklärt ist, die folgenden Axiomen genügt:
|
|
\begin{itemize}
|
|
\item Die Operation $\circ$ ist assoziativ, d.h. für alle Elemente $a,b,c\in G$ gilt $a\circ (b\circ c)=(a\circ b)\circ c$
|
|
\item Die Operation $\circ$ ist umkehrbar, d.h. zu beliebigen Elementen $a,b\in G$ sind die Gleichungen $a\circ x=b$ und $y\circ a=b$ (mit $x\in G$ und $y\in G$) lösbar.
|
|
\end{itemize}
|
|
Man nennt $G$ eine kommutative (oder abelsche) Gruppe, wenn zusätzlich noch gilt:
|
|
\begin{itemize}
|
|
\item Die Operation $\circ$ ist kommutativ, d.h. für alle $a,b\in G$ gilt $a\circ b=b\circ a$
|
|
\end{itemize}
|
|
|
|
Für die Matrizenmultiplikation von G gilt:
|
|
$G*G=\begin{pmatrix}
|
|
1*1+a*0+b*0 & 1*a+a*1+b*0 & 1*b+a*c+b*1 \\
|
|
0*1+1*0+c*0 & 0*a+1*1+c*0 & 0*b+1*c+c*1 \\
|
|
0*1+0*0+1*0 & 0*a+0*1+1*0 & 0*b+0*c+1*1
|
|
\end{pmatrix} = \begin{pmatrix}
|
|
1 & 2a & 2b+ac\\
|
|
0 & 1 & 2c\\
|
|
0 & 0 & 1
|
|
\end{pmatrix}$
|
|
|
|
Zeige dass die Einheitsmatrix Element von $G$ ist.
|
|
|
|
Zeige dass die Verknüpfung in $G$ assoziativ ist.
|
|
|
|
Begründe dass alle Matrizen in $G$ invertierbar sind. %Erinnere dich dazu daran, was die Matrixmultiplikation mit dem Rang macht.
|
|
|
|
|
|
\end{solution}
|
|
|
|
\question Markus ist politikinteressiert und möchte gerne Bundeskanzler werden. Er überlegt aber noch welcher Partei er beitritt. Er hat zwei Parteien $A$ und $B$, die ihm gefallen, könnte aber auch eine eigene Partei $C$ gründen. Die Chancen bei den nächsten Wahlen als Spitzenkandidat aufgestellt zu werden schätzt er auf $10\%$ bei Partei $A$, auf $20\%$ bei Partei $B$ und $100\%$ bei Partei $C$. Die Chance, dass die jeweilige Partei mit ihm an der Spitze die Wahl gewinnt liegt bei $60\%$, $45\%$ bzw. $2\%$.
|
|
\begin{parts}
|
|
\part Für welche Partei sollte er sich entscheiden, um mit maximaler Wahrscheinlichkeit Bundeskanzler zu werden?
|
|
\begin{solution}
|
|
S: wird Spitzenkandidat, K: wird Bundeskanzler,
|
|
|
|
$P(A \cap S) = 0,1$, $P(B\cap S)=0,2$, $P(C\cap S)=1$
|
|
|
|
$P(A\cap K)= 0,6$, $P(B\cap K)=0,45$, $P(C\cap K)=0,02$
|
|
|
|
$P_A(S\cap K) = P(A \cap S) \cap P(A \cap K) = 0,1 * 0,6 = 0,06$
|
|
|
|
$P_B(S \cap K)= P(B \cap S) \cap P(B \cap K) = 0,2 * 0,45 = 0,09$
|
|
|
|
$P_C(C \cap K)= P(C \cap S) \cap P(C \cap K)= 1 * 0,02 = 0,02$
|
|
|
|
Markus hat bei Partei B die größten Chancen Bundeskanzler zu werden (mit 9\%).
|
|
\end{solution}
|
|
|
|
\part Markus lässt die Würfel entscheiden. Bei $1$ tritt er Partei $A$ bei, bei $2$ oder $3$ Partei $B$ und bei $4,5$ oder $6$ gründet er Partei $C$. Markus wird tatsächlich Bundeskanzler. Mit welcher Wahrscheinlichkeit hat er dann Partei $C$ gegründet.
|
|
\begin{solution}
|
|
$P(\text{Tritt A bei}) = \frac{1}{6}$, $P(\text{Tritt B bei})=\frac{2}{6}=\frac{1}{3}$, $P(\text{Tritt C bei})=\frac{3}{6}=\frac{1}{2}$
|
|
|
|
$P_{\text{Tritt C bei}}(\text{C gewinnt mit ihm}) = \frac{P_C(C \cap K)}{P(\text{Tritt C bei})} = \frac{0,02}{0,5} = 0,04$
|
|
|
|
Wenn Markus Bundeskanzler wird, hat er mit 4\% Wahrscheinlichkeit seine eigene Partei C gegründet.
|
|
\end{solution}
|
|
\end{parts}
|
|
|
|
\question Gegeben sei folgender Graph:
|
|
\begin{center}
|
|
\begin{tikzpicture}[node distance = 3cm, on grid, auto]
|
|
\node (A) [state] {A};
|
|
\node (B) [state, left = of A] {B};
|
|
\node (C) [state, above left = of A] {C};
|
|
\node (D) [state, above right = of A] {D};
|
|
\node (E) [state, below left = of A] {E};
|
|
\node (F) [state, above = of A] {F};
|
|
\node (G) [state, below right = of A] {G};
|
|
\node (H) [state, right = of A] {H};
|
|
\node (I) [state, below = of A] {I};
|
|
|
|
\path [thick]
|
|
(A) edge (F)
|
|
(A) edge (D)
|
|
(A) edge (H)
|
|
(A) edge (G)
|
|
(A) edge (C)
|
|
(B) edge (C)
|
|
(B) edge (E)
|
|
(C) edge (F)
|
|
(C) edge (E)
|
|
(D) edge (H)
|
|
(E) edge (I)
|
|
(G) edge (I)
|
|
;
|
|
\end{tikzpicture}
|
|
\end{center}
|
|
\begin{parts}
|
|
\part Gebe einen Tiefensuchbaum mit Startecke $A$ für den Graphen an.
|
|
\begin{solution}
|
|
\begin{center}
|
|
\begin{tikzpicture}[node distance = 1.5cm, on grid, auto]
|
|
\node (A) [state] {A};
|
|
\node (C) [state, below left = of A] {C};
|
|
\node (D) [state, below right = of A] {D};
|
|
\node (H) [state, below = of D] {H};
|
|
\node (B) [state, below left= of C] {B};
|
|
\node (E) [state, below = of B] {E};
|
|
\node (I) [state, below = of E] {I};
|
|
\node (G) [state, below = of I] {G};
|
|
\node (F) [state, below right = of C] {F};
|
|
|
|
\path [thick]
|
|
(A) edge (D)
|
|
(A) edge (C)
|
|
(B) edge (C)
|
|
(B) edge (E)
|
|
(C) edge (F)
|
|
(D) edge (H)
|
|
(E) edge (I)
|
|
(G) edge (I)
|
|
;
|
|
\end{tikzpicture}
|
|
\end{center}
|
|
\end{solution}
|
|
|
|
\part Gebe einen Breitensuchbaum mit Startecke $A$ für den Graphen an.
|
|
\begin{solution}
|
|
\begin{center}
|
|
\begin{tikzpicture}[node distance = 1cm, on grid, auto]
|
|
\node (A) [state] {A};
|
|
\node (C) [state, below left = 2cm and 3cm of A] {C};
|
|
\node (D) [state, below right = 2cm and 2cm of A] {D};
|
|
\node (F) [state, below left = 2cm and 2cm of A] {F};
|
|
\node (G) [state, below left= 2cm and 1cm of A] {G};
|
|
\node (H) [state, below right = 2cm and 1cm of A] {H};
|
|
\node (B) [state, below left = 1cm and 1cm of C] {B};
|
|
\node (E) [state, below right= 1cm and 1cm of C] {E};
|
|
\node (I) [state, below = of G] {I};
|
|
|
|
\path [thick]
|
|
(A) edge (F)
|
|
(A) edge (D)
|
|
(A) edge (H)
|
|
(A) edge (G)
|
|
(A) edge (C)
|
|
(C) edge (B)
|
|
(C) edge (E)
|
|
(G) edge (I)
|
|
;
|
|
\end{tikzpicture}
|
|
\end{center}
|
|
\end{solution}
|
|
|
|
\part Zeige, dass für jede natürliche Zahl $k\geq 1$ gilt: Jeder Baum, der eine Ecke vom Grad $k$ enthält, hat mindestens $k$ Blätter.
|
|
\begin{solution}
|
|
|
|
Induktionsannahme: Es wird angenommen der Baum ist homogen verteilt, d.h. die Teilbäume jedes Baumes sind von gleicher Kantenlänge (Größe). Besitzt ein Teilbaum keinen Unterbaum, so ist er ein Blatt.
|
|
|
|
Induktionsstart: Für $k=1$ besitzt ein Baum maximal $2^1$ Kanten mit mindestens 1 Blatt. Daraus folgt $k=1=\sum Blätter$ stimmt
|
|
|
|
Induktionsschritt: Für $k=n+1$ besitzt ein Baum maxumal $2^{n+1}$ Kanten mit mindestens 1 Blatt. Daraus folgt $k=n+1...$
|
|
|
|
\end{solution}
|
|
\end{parts}
|
|
\end{questions}
|
|
\end{document} |